www.vorhilfe.de
- Förderverein -
Der Förderverein.

Gemeinnütziger Verein zur Finanzierung des Projekts Vorhilfe.de.
Hallo Gast!einloggen | registrieren ]
Startseite · Mitglieder · Impressum
Forenbaum
^ Forenbaum
Status VH e.V.
  Status Vereinsforum

Gezeigt werden alle Foren bis zur Tiefe 2

Navigation
 Startseite...
 Suchen
 Impressum
Das Projekt
Server und Internetanbindung werden durch Spenden finanziert.
Organisiert wird das Projekt von unserem Koordinatorenteam.
Hunderte Mitglieder helfen ehrenamtlich in unseren moderierten Foren.
Anbieter der Seite ist der gemeinnützige Verein "Vorhilfe.de e.V.".
Partnerseiten
Weitere Fächer:

Open Source FunktionenplotterFunkyPlot: Kostenloser und quelloffener Funktionenplotter für Linux und andere Betriebssysteme
Forum "Wahrscheinlichkeitstheorie" - Markovkette, Gleichgewicht
Markovkette, Gleichgewicht < Wahrscheinlichkeitstheorie < Stochastik < Hochschule < Mathe < Vorhilfe
Ansicht: [ geschachtelt ] | ^ Forum "Wahrscheinlichkeitstheorie"  | ^^ Alle Foren  | ^ Forenbaum  | Materialien

Markovkette, Gleichgewicht: Rückfrage
Status: (Frage) überfällig Status 
Datum: 13:07 So 11.02.2018
Autor: Gwenn_s

Aufgabe
[Dateianhang nicht öffentlich]
[Dateianhang nicht öffentlich]

Ich habe diese Frage in keinem Forum auf anderen Internetseiten gestellt.

Ich versuche gerade einen Teilausschnitt bzw den ersten Teil vom Beweis über die Konvergenz einer Markovkette gegen ihre Gleichgewichtsverteilung nachzuvollziehen und
ich bin ein wenig verwirrt und wollte daher noch einmal kurz nachhaken:

1) [mm] p_{(i,j)},_{(k,l)} [/mm] =  
                 [mm] P(X_n [/mm] = k | [mm] X_{n-1} [/mm] = i , [mm] Y_m [/mm] = l | [mm] Y_{m-1} [/mm] = j) =
                  [mm] P(X_n [/mm] = k | [mm] X_{n-1} [/mm] = i) * [mm] P(Y_m [/mm] = l | [mm] Y_{m-1} [/mm] = j)
    da [mm] X_n [/mm] und [mm] Y_n [/mm] unabhängig. Kann man das so interpretieren ?

2) [mm] p_{(i,j)},_{(k,l)} [/mm] auch für jede Potenz von n unabhängig,daher :

     [mm] p_{(i,j)},_{(k,l)}^{(n)} [/mm] = [mm] p_{(i,k)}^{(n)} [/mm] * [mm] p_{(j,l)}^{(n)} [/mm]

3) [mm] \pi [/mm] ist wirklich die stationäre Verteilung, denn:
   [mm] \sum_{i,j \in I} \pi(i) \pi(j) p_{(i,k)}^{(n)} p_{(j,l)}^{(n)} [/mm] =
   = [mm] \sum_{i \in I} \pi(i) p_{(i,k)}^{(n)} \sum_{j \in I} \pi(j) p_{(j,l)}^{(n)} [/mm] = [mm] \pi(k) \pi(j) [/mm]

Schon mal Vielen Dank !
  

Dateianhänge:
Anhang Nr. 1 (Typ: png) [nicht öffentlich]
Anhang Nr. 2 (Typ: png) [nicht öffentlich]
        
Bezug
Markovkette, Gleichgewicht: Fälligkeit abgelaufen
Status: (Mitteilung) Reaktion unnötig Status 
Datum: 13:20 Mi 14.02.2018
Autor: matux

$MATUXTEXT(ueberfaellige_frage)
Bezug
Ansicht: [ geschachtelt ] | ^ Forum "Wahrscheinlichkeitstheorie"  | ^^ Alle Foren  | ^ Forenbaum  | Materialien


^ Seitenanfang ^
ev.vorhilfe.de
[ Startseite | Mitglieder | Impressum ]